LSAT and Law School Admissions Forum

Get expert LSAT preparation and law school admissions advice from PowerScore Test Preparation.

 Administrator
PowerScore Staff
  • PowerScore Staff
  • Posts: 8917
  • Joined: Feb 02, 2011
|
#32013
Complete Question Explanation
(The complete setup for this game can be found here: lsat/viewtopic.php?t=15241)

The correct answer choice is (E)

If K is assigned to area 2, first consider what would happen if O is in 2 with it: that would force J and K together (closing area 2), which moves L to area 3 with M, and leaves P alone in area 1. But this violates the second rule, so we cannot have O and K together in area 2 (this is seen in Template 2). This inference alone eliminates answer choice (D).
PT79_Game_#2_#10_diagram 1.png
Next, let’s consider the assignment of L. Since L must be assigned to the same area as either K or M, it is clear that L cannot be assigned to area 1. P cannot be assigned to area 1 either (in compliance with the second rule). Accordingly, J is the only ranger that can—and therefore must—be assigned to area 1:
PT79_Game_#2_#10_diagram 2.png
P and L then are the only rangers whose assignments are not fully determined. This uncertainty is crucial in answering a Could Be True question, as it instantly narrows the range of possible contenders to answer choices (C) and (E). Answer choice (C) is clearly impossible, because L cannot be assigned to area 1 (see above). Therefore, answer choice (E) is correct.
You do not have the required permissions to view the files attached to this post.

Get the most out of your LSAT Prep Plus subscription.

Analyze and track your performance with our Testing and Analytics Package.